« first day (29 days earlier)      last day (545 days later) » 
00:00 - 16:0016:00 - 00:00

4:01 PM
0
Q: Why are translates of travelling waves again travelling waves?

M. MeyerA travelling wave solution of a PDE or ODE is a solution that depends on the single variable $\xi=x-ct$. For example consider the PDE $$ u_t=u_{xx}+f(u)-w,~~~w_t=\epsilon (u-\gamma w).~~~~~(1) $$ Then, a travelling wave $(u(\xi), w(\xi)$ satisfies $$ -cu_{\xi}=u_{\xi\xi}+f(u)-w,~~~~~-cw_{\xi}=\...

0
Q: Finite automata language question review

dmnteI have completed the questions below but am not sure if they are correct. If anyone could help me confirm them it would be much appreciated. 3) This took me a little while but it seems to hold up. Im not sure if just writing (n+m) = odd number is ok but they use something similar in the fourth q...

0
Q: Proof of prime numbers in the form..

RexThere is a prime that is unique in the form of p^2 -1, p is just some integer with the restriction of p being greater than or equal to 2. Prove this. I understand that I am first suppose show a prime p exists and that another p' exists then p=p'.But I am quite confused.

Words such as question do not add information to titles. Please edit the title so that it better describes the specifics of your question. Do not hesitate to make it longer or include a formula if needed. More tips here. (from a bot)Normal Human 20 secs ago
0
Q: Congruence Classes

the manI've been learning about congruence classes recently and have been having some trouble understanding the following fact; $$[a]_m = {\{b \in \mathbb{Z} : a \equiv b(modm)}\}$$ Now, these are obviously all $b$ of the form $b = a-mk, k \in \mathbb{Z}$ Now I have trouble understanding the set of al...

Short title. Congruence Classes
0
Q: If $[x]^2-5[x]+6=0$,where $[x]$ denotes the greatest integer less than or equal to $x$,then total set of values of $x$ is

Vinod Kumar PuniaIf $[x]^2-5[x]+6=0$,where $[x]$ denotes the greatest integer less than or equal to $x$,then total set of values of $x$ is $(A)x\in[3,4)$$(B)x\in[2,3]$$(C)x\in\left\{2,3\right\}$$(D)x\in[2,4)$ My attempt: $[x]^2-5[x]+6=0$ $[x]=2,[x]=3$ $2\leq x<3,3\leq x<4$ But i cant figure out what is the ans...

Question contains please. [If $[x]^2-5[x]+6=0$,where $[x]$ denotes the greatest integer less than or equal to $x$,then total set of values of $x$ is](math.stackexchange.com/q/1554998)
0
Q: If $S $ is compact then $S$ is closed.

Angelo MarkMy proof : Let $x \in S'$ Suppose $x \notin S$. Therefore $B(x,r) \cap S =\phi$ for all $r>0.$ This a contradiction as $x \in S' \Rightarrow\exists r>0 $ s.t. $B(x,r)/\{x\}\cap S \neq\phi$ So $x \in S$ $\therefore S'\subseteq S $ So $S=\bar S=S \cup S'$. Thus $S$ is closed.

Tagged proof-verification. If $S $ is compact then $S$ is closed.
0
Q: Definite integral $\int_{\frac{-\pi}{4}}^{\frac{\pi}{4}}\ln(\cos x + \sin x) dx$

Kay K.Evaluate the following integral: $$\int_{\frac{-\pi}{4}}^{\frac{\pi}{4}}\ln(\cos x + \sin x) dx$$

0
Q: Calc 3 Workshop

Chris RabeonyYou are standing on the floor of a cave, observing the wildlife. By good fortune, the cave is in the shape of a hemisphere with height 84 feet. a) A bat flies in and around the upper reaches of the cave, always staying at least 28 feet above the floor of the cave, because bats are very superstit...

Short title. Calc 3 Workshop
0
Q: Norms- Separation of points

JackHow do I show separation of points for (1) $\|f\|_\infty +\|f\|_1$ My answer: $\|f\|_\infty = 0 \implies f=0$ and $\|f\|_1 \implies f=0$ so $\|f\|_\infty +\|f\|_1=0 \implies f=0+0=0$ because I know $\|f\|_\infty $ and $\|f\|_1$ are norms. And $f=0 \implies \|f\|_\infty=0 $ and $f=0 \implies \|...

 
4:25 PM
0
Q: Policy on syntax clarification questions

Brian CampbellIs there a policy, or a canonical thread on discussion policy, on syntax clarification questions? This question on an obscure piece of Ruby regex syntax is the motivation for this question. It asks about what (?-mix:) around a regular expression means. This isn't a very common piece of syntax, a...

 
0
Q: Is this simplification of boolean algebra correct?

BrandonCW = A`.B.C`.D` + A.B`.C`.D` + A.B`.C.D` + A.B.C`.D + A.B.C.D` CW = A`.B.C`.D` + A.B`.D`(C`+C) + A.B(C`.D + C.D`) CW = A`.B.C`.D` + A.B`.D`(1) + A.B(C`.D +D`.C) CW = A`.B.C`.D` + A.B`.D` + A.B(C`.1.C) CW = A`.B.C`.D` + A.B`.D` + A.B(1) CW = A`.B.C`.D` + A.B`.D` + A.B CW = A.B + A`.B.C`.D` + A.B`....

0
Q: Difference between A+B and A ∪B

k.dkhkBasic set theory: A and B are two sets. I assume that A+B isn't the same as A∪B. I know what A∪B is but what is A+B

0
Q: Definitions On Landau Notation (Big O and little o)

Manolis LyviakisWhat are the definitions on Big O and little o for when $x \in R^m$ approaches $s\in R^m$ And not $x$ going to infinity?

0
Q: Find the ordinary generating function with coefficients

chocolate123Find the ordinary generating function with coefficients ak equal to the number of ways we can distribute k pieces of candy to n children such that no child gets more than m pieces.

0
Q: Graph theory max number of components

JohnWhat is the highest count of components this graph can have? (1,2,2,2,2,2,2,2,2,2,2,3,4,5,5) I am not really sure how to compute this, can anyone hint/help me?

0
Q: Describe the centralizer

SelfStudyExercise from Artin's Algebra. Describe the centralizer $Z (\sigma) $ of the permutation $\sigma = (153)(246)$ in the symmetric group $S_{7} $, and compute the orders of $Z (\sigma) $ and of $Z (\sigma)$. My progress: Since $P= 1/\sqrt {2} \begin {pmatrix} 1 & i \\ 1 & -I \end {pmatrix}$ has...

 
4:52 PM
0
Q: What's wrong with the rep graph tab?

MickGI just found out the reputation tab on the profile page has this graph feature. I looked at the small graph and saw a high peak in September. I tried to view it by clicking and dragging, but was confronted with a huge peak and no info on what caused it: Now this suggested either a huge loss to...

 
0
Q: Existence of pullback tensor

MathStarterIf $M,N$ are smooth manifolds and $F: M \to N$ is a surjective smooth submersion. A tangent vector $v \in T_p M $ is called vertical if $d F_p (v) = 0$. Now suppose $\omega \in \Omega^k (M)$, I want to show that if $v \lrcorner \omega _p =0 $ and $v \lrcorner d \omega _p = 0$ for all $p \in M$ an...

0
Q: this might be a bit easy for you, what is the method to solve these kind of questions?

SadStudentx and y are positive integers, 43! = 3^x.y What is the greatest value that x can take?

0
Q: Hessian-Matrix of a Determinant

AlinaLet $f:\mathbb{R}^{n}\rightarrow \mathbb{R}$ be a polynomial function and let $H_f(x)$ denote its Hessian. Now define $p:=det(H_f(x))$. Is there a noce way to relate the Hessian of p i.e. $H_p(x)$ with the Hessian of $f$?

Consider adding a tag for a broader subject area to which the question belongs. Some of these tags might fit. (from a bot)Normal Human 21 secs ago
0
Q: Neighborhoods: Generator

Freeze_SContext In topological groups it is more convenient to work with neighborhoods instead of opens. In this respect one may characterize the topology by neighborhoods of the origin. Moreover local compactness and other local concepts become straightforward this way. For a comparison see the wikiped...

0
Q: Given three side lengths, how many triangles can be formed

user294975Given three side lengths, how many triangles can be formed. A. 1 B. 2 C. 3 D. infinitely many

0
Q: Quick question about Rubik's Cube with a missing color

Shrey AryanSuppose a color of the Rubik's cube(3x3) is missing, is it possible to find the missing color? Now a brute force method would be solve for all other colors, but that is quite naive.

0
Q: Showing that an equation has no solution in $\Bbb Z$

NidShow that $x^3 + 10x^2 + 6x + 2 = 0$ has no solutions in $\Bbb Z$. This seems rather trivial to do but I don't know how to rigorously show this is true. Having graphed this and attempted to factor,I see that it is indeed true. Could someone please explain how I would go about showing this rigoro...

Words such as question are uninformative in titles. Please edit the title so that it better describes the specifics of your question. Do not hesitate to make it longer or include a formula if needed. More tips here. (autocomment)Normal Human 21 secs ago
0
Q: How to start with finding the extremes of the multi-variable function?

Andrej Should i start with G(x,y,z,lambda) ? function: f(x, y, z) = xyz condtions: x + y + z = 5 and xy + xz + yz = 8.

 
5:10 PM
0
Q: How are acceptable audit answers decided?

Brian RayI stumbled across this question while reviewing first posts. It essentially has 2 parts, how to organize client-side user authentication code the Redux way and how to best integrate Facebook login into Redux. I flagged this as "low quality" since the first part should be a simple Google search an...

 
0
Q: Limit of question no 4

Subhash Kumar sharmaHi, please evaluate the limit of the function depicted in the image.

Welcome to Math.SE, Subhash Kumar sharma. Words such as question are uninformative in titles. Please edit the title so that it better describes the specifics of your question. Do not hesitate to make it longer or include a formula if needed. More tips here. (autocomment)Normal Human 21 secs ago
0
Q: Prove, by logical reasoning, rather than by formula the following permutation identities

Gaurang TandonThe formula would have been useful but I am not really good at logical reasoning especially in permutations so I need help from you guys to identify errors in my answer (as well as give hints for part b and e) (a) $n_r=(n-1)_r+r(n-1)_{r-1}$ $n_r$ is the number of ways to choose $r$ objects...

0
Q: Legendre polynomial

Master MathematicianHow\ Can\ I\ prove\ that : P_{n}(1)=\frac{\left ( \frac{1}{2} \right ){n}2^{n}}{n!} {2}F{1}(\frac{1}{2}-\frac{n}{2},\frac{-n}{2};\frac{1}{2}-n;1)=1 \ using \ gauss\ '\ theorem : \ \ 2F_{1}(\alpha ,\beta ;\gamma ;1)=\frac{\Gamma (\gamma )\Gamma (\gamma -\alpha -\beta )}{\Gamma (\gamma -\alpha...

This site uses MathJax formatting of formulas. Consider adding a tag for a broader subject area to which the question belongs. Some of these tags might fit. More tips here. (from a bot)Normal Human 21 secs ago
 
0
Q: Why the reputation changed without notification.

SolitaryLast night my reputation was $5199$, and this morning it became $5164$, without any notification under the reputation tag. This question was asked before. But I just want to make sure where can I view how reputation changes (clearly, the usual reputation tag doesn't help). Earlier I got notific...

 
5:28 PM
0
Q: Looking for someone to help check over probability questions

QualityHi I am prerparing for exam and am doing old exams from other schools and places online , in particular questions that to me cover the same stuff we have. However usually I don't find any solutions. So I thought I will try them and post and see if anyone can please tell me if I am wrong or not et...

Title contains someone, help. Tagged proof-verification. Looking for someone to help check over probability questions
0
Q: Degenerate mapping.

Kasia PisaniakWe say, that mapping bilinear $ \phi $ is degenerate if $(\forall _y \phi (x,y)=0\implies x=0$. Prove, that $\phi$ is degenerate $\iff det(\phi(e_i,e_j))\neq0$. Could you help me? I don't no how I must do this. I'm sorry for my English, is not good.

Short title. Degenerate mapping.
0
Q: How to andle this curve?

Andrej I started with diferentiation of all three coordinates of this paramtetrically given curve. I want to show that the respective curve has related equation of the plane and also to proove that it a "plane-curve".

0
Q: Geometric meaning of the expression

J.ExactorWhat is the geometric meaning of the following expression: $$ \dfrac{x^n + y^n}{2} > \left(\dfrac{x+y}{2}\right)^n$$

Consider replacing (analysis) with a more specific tag for the relevant branch of analysis. (from a bot)Normal Human 21 secs ago
0
Q: How can I explain my logic? - Related to Herfindahl index

Yongjin KwonI've tried to measure something that I have in mind. My problem is as following: Let's assume that there is a group with 8 members. There are two cases: First, A group consists of 4 subgroups each with 1,1,2, and 4 members.(1+1+2+4=8) Second, A group consists of 4 subgroups each with 2 members...

Welcome to Math.SE, Yongjin Kwon. This site uses MathJax formatting of formulas. More tips here. (from a bot)Normal Human 21 secs ago
0
Q: Density function evaluation probability

PersonaACan anyone help to tell me if this is how you would do the following; I calculated that $k=6$, for b I got $0.7407$, and for c I got $E[Y]=0.5$ while for d I got $V[Y]=0.05$ But I dont know if I am did the question correct or completely wrong. Would anyone mind please letting me know?

Question contains please. Density function evaluation probability
 
5:43 PM
0
Q: Spam answer as Low-Quality-review test?

Byte CommanderI just had to review this test in the LQ queue, which was a spam answer. However, my thought was that spam should never had appeared in the LQ queue and therefore I was a bit confused about this. I recommended to delete and passed the test, of course, but I thought that maybe this could be an un...

 
0
Q: How can $(\Bbb{Z}/(3) \times \Bbb{Z}/(5))^{\times} \approx \Bbb{Z}/(2) \times \Bbb{Z}/(4) \approx \Bbb{Z}/(16)$?

Enjoys MathSee the table on this page.. That would mean $\Bbb{Z}/(15)$ has more than $15$ elements, so I've missed something. Thanks.

0
Q: modulus function doubt

Praveen Kumarmaximum of of 2|x|+|x+2|-||x+2|-2|x|| how to solve this using graph. explain it graph

Welcome to Math.SE, Praveen Kumar. Words such as doubt are uninformative in titles. Please edit the title so that it better describes the specifics of your question. Do not hesitate to make it longer or include a formula if needed. More tips here. (from a bot)Normal Human 21 secs ago
 
-7
Q: Why would this answer be given a minus?

Daniel CortesI recently tried to contribute to SO by answering a question regarding what are the components of Rails. I noticed a lot of people gave very complicated answers and received lots of points for it. This is the link to the question asked by the candidate: What exactly are the "components" of Rails...

0
Q: Bug with comments display

idmHave a look at this question Moreover, I can't delete my comment because the delete button overlaps with the sidebar... Also, if I try to add another comment, the line break is still not working well.

0
Q: Potential bug in body validation pop-up

halferJust now I tried to edit this question to remove the urgent begging, so I took out this line: Urgently waiting for a solution, thanks in advance. When I did so, even prior to submitting the edit, a pop-up complained about the first line, saying: Body cannot contain "I encounte" Hmm, I...

 
6:02 PM
0
Q: find $y'$ for $y=(4+x^2)^x$

Ilia LabkovskyThis differentiation requires the use of natural logarithms (the laws of logarithms), differentiation of logarithms, exponential function differentiation and the power rule. the formula for differentiation of exponential functions is $d/dxa^x = a^x*ln(a)$ I use this to get $dy/dx = (4+x^2)^x*ln...

Question contains please. find $y'$ for $y=(4+x^2)^x$
0
Q: Can I place the rubiks3cube pieces in the distorted position I intend to get?

supremumRight now I am trying to get the distorted position like this: in each face only one diagonal is solved and no similar colour is on a face other than the diagonal pieces mentioned previously. For example: If I am looking at the white face, there should exactly be two corner pieces in that face co...

Questions tend to get more attention when they have a tag for a broad area of mathematics relevant to the question. Some of these tags might fit. (autocomment)Normal Human 21 secs ago
0
Q: Simple Normed Space Question

Anthony PeterLet $(V,\|\cdot\|)$ be a normed vector space. Let $x,y,x',y' \in V$. Say I want to estimate $$\left| \|x\|-\|x'\|-(\|y\| - \|y'\|) \right|.$$ Does the following chain of inequalities hold?: \begin{eqnarray*} \left| \|x\|-\|x'\|-(\|y\| - \|y'\|) \right| &\leq& \left| \|x-x'\|-(\|y\| - \|y'...

Words such as question are uninformative in titles. Please edit the title so that it better describes the specifics of your question. Do not hesitate to make it longer or include a formula if needed. More tips here. (from a bot)Normal Human 21 secs ago
0
Q: Finding the number of non negative integer solutions to an inequality with an upper bound

KevinHow many non-negative integer solutions are there to the equation x1 + x2 + x3 + x4 + x5 < 11, (i)if there are no restrictions? (ii)How many solutions are there if x1 > 3? (iii)How many solutions are there if each xi < 3? (i) inequality equivalent to equality x1 + x2 + x3 + x4 + x5 + x6 = 10 (n...

0
Q: How should I approach this Poisson distribution problem?

user3699627Show that the Poisson probabilities $ 𝑃𝜇(𝑘)$ satisfy the recurrence relation $𝑃𝜇(𝑘)=𝜇𝑘𝑃𝜇(𝑘−1)$ and hence determine the values k, for which the terms $𝑃𝜇(𝑘)$ reach their maximum for given $𝜇 $.

Welcome to Math.SE, Kevin. This site uses MathJax formatting of formulas. More tips here. (from a bot)Normal Human 21 secs ago
0
Q: How the tackle this limited integral?

Algebra 2015I started with use of a new variable for the things under square root. I would like to calculate the integral over the respected area.

0
Q: Useful summation for rmt.

Aditya KumarI encountered the following sum while solving a problem using Ramanujan's master theorem. Please prove: $$\frac { sin\left( 2t\left( arctan\sqrt { x } \right) \right) }{ \sqrt { x } { \left( 1+x \right) }^{ t } } =\sum _{ k=0 }^{ \infty }{ \frac { \Gamma \left( 2k+2t+1 \right) \Gamma \left...

Short title. Question contains please. Useful summation for rmt.
0
Q: Find the area of the region that lies inside the first curve and outside the second curve. r = 10 cos θ, r = 5

SunnyI am not sure of my answer. In the figure, r=10 sin(theta) is a circle that doesn't look like a circle. The area of r=5 is pi r^2 = 25 pi You remove the area from -pi/3 to pi/3 of 10 cos(theta) from 25 pi That is remove (1/2) ∫ (10 cos(theta))^2 dtheta = 74.0105 Required area = 25 pi - 74.0...

This site uses MathJax formatting of formulas. More tips here. (autocomment)Normal Human 21 secs ago
0
Q: Let P4 be the vector space of polynomials of degree at most 4. For the following map decide if it is an isomorphism?

atakanHow can i describe this operation as an isomorphism or not ? p(x)=p_0+(p_1)x+(p_2)x^2+(p_3)x^3+(p_4)x^4>>>q(x)=p_1+(p_2)x+(p_3)x^2+(p_4)x^3+(p_0)x^4

Welcome to Math.SE, atakan. This site uses MathJax formatting of formulas. More tips here. (from a bot)Normal Human 21 secs ago
0
Q: Invariant Ring vs Polynomial Ring

NinjaCan someone explain these two structures as widely as possible? I consider a group $G$ which has finite representation of dimension $n$ with field $K$.

0
Q: Is it possible to prove (using the pumping lemma) that the language L = {w | w has even length and the second half of w does not contain any 0s}

geek4079I'm struggling with finding a starting string $s$ to prove that language L is not regular. Any suggestions? Thanks

0
Q: How to use the gamma function at this question?

user243179I started wit use of polar coordinates. I have probles at with help of gama function to calculate the volume of the geometric body limited by 2 curves http://www4.slikomat.com/13/1201/nxd-geomet.jpg http://www4.slikomat.com/13/1201/u51-geomet.jpg when x>= 0

Words such as question are uninformative in titles. Please edit the title so that it better describes the specifics of your question. Do not hesitate to make it longer or include a formula if needed. More tips here. (from a bot)Normal Human 21 secs ago
0
Q: Help finding a 10 parameter equivalent to a 5 parameter fitting function?

RicoRicoThis is my first question posted here. I apologize if this post is misformatted or inappropriate in any way. I have a relatively simple fitting function f(m): $$f(m) = {1\over\sqrt{{A\over M_1} + {BV_1\over m}}} + C$$ where $A, B, C, M_1$ and $ V_1 $ are known fitting parameters for a particul...

 
6:36 PM
-5
Q: Grammar error on the user profile page

Ericson WilliansEach time I open a profile with an empty description, I read: Apparently, this user prefers to keep an air of mystery about them. The correct would not be Apparently, this user prefers to keep an air of mystery about him/her. ? Why them if it's not plural? This annoys me each time, ...

 
0
Q: General question about pumping lemma for regular languages

JohnAccording to the formal statement of the lemma here: https://en.wikipedia.org/wiki/Pumping_lemma_for_regular_languages It is written at (3) that for all $i≥0, xy^iz∈L$. Until this moment, I was certain that $i$ must be a natural number. But what if, for example, $|y|=4$ and I wa...

Words such as question are uninformative in titles. Please edit the title so that it better describes the specifics of your question. Do not hesitate to make it longer or include a formula if needed. More tips here. (autocomment)Normal Human 21 secs ago
0
Q: Prove inequality statements

user2850514Given the three inequalities: \begin{align} a&<0\\ b&<0\\ c+d&<0 \end{align} Are the conditions below satisfied? Justify your answer. $a+b<0$ $ab-cd>0$ $\alpha a + b>0$ $(\alpha a + b)^2 > 4\alpha(ab-cd)$ True by adding the first two given inequalities. From the first two inequalities we...

0
Q: Every finite dimensional Self-Adjoint Linear Map has an orthonormal basis of eigenvectors Proof

Arcane1729 What I understand: By dimension formula $dim V$ $-$ $1$= $dim \langle v \rangle ^ \perp$ since $V$ is the direct sum of $U$ and $U ^ \perp$ if $U$ is a subspace of $V$. dim $\langle v \rangle$ is clearly $1$ So it makes sense that the number of basis vectors for $V$ is $1$ more than the numbe...

0
Q: Fill graph sequence

user288083I have got not increasing sequence: $6,x,4,4,4,4,3,2$. And I have task, fill the sequence so that the sequence of some graph. Please can you hint me, how to do it?

Short title. Short question. Question contains please. Fill graph sequence
0
Q: The homology of wedge sum

Aloizio MacedoThis is an exercise of Bredon (pg. 190) which I tried to do but got stuck at one part. He asks the following: Let $X$ be a Hausdorff space and let $x_0 \in X$ be a point having a closed neighbourhood $N$ in $X$, of which $\{x_0\}$ is a strong deformation retract. Let $Y$ be a Hausdorff space ...

 
0
Q: How do I proceed when a question received many answers, none satisfactory?

ViclibI've asked this question. It has received 3 very informative answers that I read, understood and learned a lot from. But they do not answer the main motive of the question, which is implementing modular web components. They all implement web components, but which are not modular, because you can'...

 
6:54 PM
0
Q: How do you find $y'$ for $x^y = y^x$?

Ilia LabkovskyUsing the laws of logarithms: $yln(x) = xln(y)$, $y = xln(y)/ln(x)$ Is it now quotient rule for the derivative? How is this done?

0
Q: Regular expressions divisible by 3

VaroagGive a regular expression for the language L over Σ = {a, b}* of words that contain a number of b’s that is evenly divisible by 3. I know that this expression: $(a^∗ba^∗ba^∗b)^∗a^∗$ works for the alphabet {a,b}, but the asterisk changes things. I would like to know how.

Title ends with a digit. Regular expressions divisible by 3
0
Q: Set theory problem cartesian multiplication problem

Itamar SilversteinProov or give counterexample: (AXB) - (CXD) = (A-C)X(B-D)

Title contains problem, problem. Short question. Set theory problem cartesian multiplication problem
0
Q: On the propertics on triangles

A.GLet $A_{1}=A_{2}$ and $BM=MC$. Then prove that $AB=AC$. Thank you

0
Q: Gluing the Half-planes

Anthony BrittoA homework question asks us to show that $H_+ \cup_{\mathrm{id}_{\partial H_+}}H_-$ is homeomorphic to $\mathbb{R}^n$, where $H_+ := \{x \in \mathbb{R}^n\ |\ x_n \geq 0\}$ is the upper half Euclidean plane, $H_-$, defined similarly, is the lower half plane, and the gluing operation is defined as ...

0
Q: Highest count of components of graph

JohnWhat is the highest count of components this non-decreasing simple graph can have? (1,2,2,2,2,2,2,2,2,2,2,3,4,5,5)

0
Q: Linear algebra- Write 1+i in polar form,then use this to compute (1+i)^10

NickI understand how to write 1+i in polar form, but how do I use it to compute (1+i)^10? Thanks for the help!

0
Q: minimize tr($(G^TG)^{-1}$)

user2923937I have been trying to minimize tr($(G^TG)^{-1}$) using cvx. I have formulated it in the following SDP structure, using Schur Complement. Here is the formulation: $$\mathbf{minimise} \ \ t \\\mathbf{subject\ \ to}$$ $$\begin{bmatrix} I & G \\G^T & -X \end{bmatrix} \succeq 0 \qquad \begin{bmatrix} Z

0
Q: Algebra - Automorphism, proof

vannyhow to show that map σ((a,1),(1,b)) = ((a^3,1),(a^2,b)) is an automorphism? Thank You.

Short title. Short question. Algebra - Automorphism, proof
 
7:21 PM
0
Q: What is the difference between two duplicate flag options? What is the necessity of these two?

manetsusWhen I go to the flag any question, I get two options for flagging as duplicate. One is a duplicate...: the other is duplicate of...: There even no difference in there short description. From intuition, as far I understand that the first one means "This question is a duplicate question of...

 
0
Q: Question about a divisibility rule

SadStudentA < B 7A3B/12=m.k+0 How many different values are there that A can take?

Words such as question are uninformative in titles. Please edit the title so that it better describes the specifics of your question. Do not hesitate to make it longer or include a formula if needed. More tips here. (autocomment)Normal Human 21 secs ago
0
Q: How do I prove that $f(x) = ln(x) − (x − 4)^2$ has exactly two roots.

Ilia LabkovskyI assume we take the derivative of the function. I get: $y' = 1/x-2(x-4)$ and I attempt to set it to 0 and solve but get stuck. Any tips?

0
Q: Scaling and Rotation Matrices

jimmy talonWrite the Matrix \begin{bmatrix}1&-1\\1&1\end{bmatrix} as the product of a scaling matrix with factor |λ| and a rotation matrix with angle φ. Find |λ| and φ for which −π < φ < π. How would I express this matrix as a scaling and rotation matrix in terms of λ and φ? I'm not sure how to approach th...

0
Q: 2 problems related to the number 2015

András Szilveszter Let $p=\underbrace{11\cdots1}_\text{2015}\underbrace{22\cdots2}_\text{2015}$. Find $n$, where $n(n+1) = p$ Prove that $\frac{1}{2^2} + \frac{1}{3^2} + \cdots + \frac{1}{2015^2} < \frac{2014}{2015}$ For 1, I tried dividing in various ways until I got a simpler expression, but no result. For 2,...

0
Q: What is the integral of (x-1)/[(x+3)(x^2+1)]?

CallumI've worked with partial fractions to get the integral in the form A/(x+3) + (Bx + C)/(x^2+1). Is there a quicker way?

This site uses MathJax formatting of formulas. More tips here. (autocomment)Normal Human 21 secs ago
0
Q: Holomorphic Transformation

J.R.Let $f$ $\ne$ const be holomorphic in $D$ $\subset$ $C$ is open and connected. Prove that the image $f(D)$ is open and connected. Is it sufficient to prove that for any point, the image $f(D)$ contains all point of some neighborhood?

 
7:45 PM
0
Q: Why do I get a warning when I delete my own post with only my comment in it?

SadStudentI asked a question 10 minutes ago, I added a comment for more info about the question with a comment. 2 minutes later I was able to solve the problem and wanted to delete my post. I got a warning which was mentioning about a "ban" if I keep deleting my posts with comments. Is there a reason for...

 
0
Q: Invertible Matrices

Jeff DahmFind an invertible matrix P and a matrix C such that A = PCP^−1, where the matrix A is given by \begin{bmatrix}-1&-5\\4&7\end{bmatrix}. How would I find P and C, by only knowing the values of A? Any help would be appreciated.

Short title. Invertible Matrices
0
Q: continuous function bound problem

ashSuppose f is continuous on [a,b], show that there exist points x1, x2∈[a,b], such that the range of f on [a,b] is equal to [f(x1),f(x2)] I know it is true intuitively, but don't know how to prove it formally. I know that f is continuous on [a,b] and therefore f is uniformly continuous , then I t...

Title contains problem. continuous function bound problem
0
Q: Number of operations required for gaussian elimination of tridiagonal matrix

JanetHorneHow do I account for (or rather, not account for) the 0's in the matrix so I don't do more operations than necessary? Thanks.

0
Q: A problem for MAthematical Analysis

EklavyaLet $X$ bea proper closed subset of $[0,1]$ .which of following statements is always true? $A)$the set $X$ is countable $B)$the set $X$ contain an open interval $C)$ the exist $x$ $\in$$X$ such that $X$/{x} is closed $D)$none of above

Title contains problem. A problem for MAthematical Analysis
 
-3
Q: Mixed messages: topic marked as closed, but voting still works

Nora McDougall-CollinsBackground: A business acquaintance recently asked me to look at his site. I found that the site was built with SiteGrinder, which I had never heard of. I was hoping for technical comments from the StackOverflow community about this software. My search for SiteGrinder lead me to a Question that...

-6
Q: Ask a question direct to another user

physics90I recently tried posting a question to an old post hoping the original author would answer my question since he/she had created the same project previously. My question was downgraded because it was new. Is there anyway to ask or tag another user in your question?

0
Q: Is this user being too noisy?

blackThis user has a page or more of approved edits - that carry on in the proposed edit review queue-, as you may see here, which involve fixing always the same "noisy" issue: thanks found in the questions. It seems very suspicious to me: what's going on?

 
0
Q: Estimators and Variance - Uni Stats and Maths

John BrownAny help with Q2)b,c and Q3 appreciated

0
Q: Prove congrunce rule

CarefullcarsSay you have the numbers a and b if a is congruent with b modulo 5, we know that a mod 5 = b mod 5. let a mod 5 = c and x,y be some whole numbers then we have a = 5x + c and b = 5y + c so a-b = 5x + c -(5y + c) = 5(x-y) 5 is a factor in a-b. Now my question is, if we start with only knowing t...

Short title. Tagged proof-explanation. Prove congrunce rule
0
Q: Find the real part and imaginary part of z= e^2-πi/3

TreshonHaving troubles solving this one because I'm not sure how to go by solving with e.

0
Q: basic inequality question

user2850514Given the three inequalities: \begin{align} bc &< -2\\ d &> 2\\ (d-2)^2 &> 4d(-2-bc) \end{align} Show that $b$ and $c$ must have opposite signs. I'm not really sure where to start here..

Words such as question do not add information to titles. Please edit the title so that it better describes the specifics of your question. Do not hesitate to make it longer or include a formula if needed. More tips here. (autocomment)Normal Human 22 secs ago
0
Q: Most notable problems in nocommutative algebraic geometry

Stuf kooriWhat are the most notable open problems in the field of noncommutative algebraic geometry?

Welcome to Math.SE, Stuf koori. Consider adding a tag for a broader subject area to which the question belongs. Some of these tags might fit. (autocomment)Normal Human 22 secs ago
0
Q: How to estimate numbers like $(19/20)^{30}$

ropIs it possible to estimate by hand what is the value of expresion like $(19/20)^{30}$? $$19/20 = 0.95$$ but $$(19/20)^{30} \approx 0.2146$$ So it is totally different number.

0
Q: Intergral of cos(x)e^(3x-1)

LeoI have a question which says to integrate $\cos(x)e^{2x-1}$. My attempt: $$\int cos(x)e^{2x-1}\, dx = \sin(x)e^{2x-1} - \int \sin(x)2e^{2x-1}\, dx $$ And, $$ \int \sin(x)2e^{2x-1}\, dx = -2cos(x)e^{2x-1} - \int -4\cos(x)e^{2x-1}\, dx$$ So,$$\int cos(x)e^{2x-1}\, dx = \sin(x)e^{2x-1} + 2\cos(x)...

0
Q: Trees in graph theory

B SmithHow many distinct trees can be constructed using 3 nodes, 4 nodes , 5 nodes and 10 nodes? i was thinking there was 3 distinct trees in 3 nodes , but am not sure about the rest.

Short title. Short question. Trees in graph theory
0
Q: Roll a fair 6 sided die twice, What is the probability that one or both rolls are 6?

BNSlugRoll a fair 6 sided die twice, What is the probability that one or both rolls are 6? This question is confusing me on a conceptual level, specifically the "or both rolls." With out that, the question is what is the probability of getting a 6 by rolling a die twice, which is of course 11/36. Th...

0
Q: If $\lim a_n = L$, then $\lim s_n = L$

JohnIf $\lim a_n = L$, then $\lim s_n = L$, where $s_n = \frac{ a_1 + \dots + a_n}{n}$

This site uses MathJax formatting of formulas. More tips here. (autocomment)Normal Human 21 secs ago
0
Q: complex numbers inequality

prometheus21Let $z\in \mathbb{C}$ such that |z|=1. Prove that $|1+z|+|1+z^2|+|1+z^3|\geq2$ I have tried using the triangle inequality by grouping the first and last module after expending $|1+z^3|$ and then grouping it all with $|1+z^2|$. I arrived at $$|z^3+z^2+z+1+2|=|\frac{1-z^4}{1-z}+2|$$ How do I prov...

0
Q: Why is this a senseful notion of stability?

M. MeyerLet $\xi=x-ct$. Moreover, let $U(\xi)$ be a travelling wave solution of a PDE. Suppose that $U(\xi,t)$ is a solution of a PDE. The travelling wave $U(\xi)$ is called stable (with respect to the PDE) if there is a neighborhood $N$ of it, such that for a solution $U(\xi,t)$ whose initial value $U...

0
Q: If $C_G(x) \leq H$ for every $p$-element $x \in H$, then $p$ cannot divide both $|H|$ and $|G:H|$

BungoThis is problem 1.D.2 in Isaacs, Finite Group Theory. I am self-studying, so would appreciate a proof verification. Note: in this book, all groups are assumed finite unless otherwise stated. Fix a prime $p$, and suppose that a subgroup $H \leq G$ has the property that $C_G(x) \leq H$ for eve...

0
Q: $b w_i = \sum\limits_{i=1}^r a_{i,j} w_j \implies \det(bI - (a_{i,j})) w_i = 0$ in an $R$-module.

Enjoys MathLet $R$ be a commutative ring with $1$ and $M$ a finitely generated $R$ module. Then if $\{w_j : j = 1 \dots r\}$ is a set of generators for $M$ and $b \in R$, then $b w_i = \sum\limits_{j=1}^r a_{i,j} w_j$ for some $a_{i,j} \in R$. This implies $\det(bI - (a_{i,j}))w_i = 0, i = 1 \dots r$ some...

0
Q: Examples of 3d visual proofs

MaesumiI am looking for examples of three dimensional constructible proofs. By this I mean activities such as steps in proving $1^2+2^2+\cdots+n^2=n(n+1)(2n+1)/6$. In this construction the identity is proven by assembling 6 special pyramids that interlock to make a box.

Short title. Tagged algebraic-identities. Examples of 3d visual proofs
0
Q: Combinatorial problem. No. of combination with repetition.

AhmoI have little task to slove. Problem is that I have 29 balls and I must distribute them into 4 boxes. An additional requirement is that each box can have max 7 balls. I think that this problem represent number of combination with repetiton, but this additional requirement makes me problem.

0
Q: Simplify the complex expression to complex numbers (2i)^i

TreshonI am getting stuck with this one.... This is how far I got 2i = 2 exp [ (i π/2 + 2kπ ) ] (2i)^ I = (sqrt 2)^ i I'm having troubles moving past that point

This site uses MathJax formatting of formulas. More tips here. (autocomment)Normal Human 21 secs ago
0
Q: Help differentiating function with norm

Peter Calvo Let E be a vectorial space of finite dimension. $$f:E^*\rightarrow E^*$$ $$x\rightarrow \frac{x}{<x,x>}$$ Is this function $C^1$? How should I proceed? My guess is evaluate $f(x+h)-f(x)$ for a small increment $h$ and obtain the differential, then check continuity. But then I start to o...

Words such as help are uninformative in titles. Please edit the title so that it better describes the specifics of your question. Do not hesitate to make it longer or include a formula if needed. More tips here. (from a bot)Normal Human 21 secs ago
0
Q: Hint for integrating exp(x-x^2)

SusliksThe function $e^{x-x^2}$ is zero if $x \to \infty$ or $x \to -\infty$ it looks like a normal-distribution-curve with the max. value at $x=0.5$. Now how can i integrate it? Thank you.

0
Q: On quiver conjunction?

Javier AriasI would be interested in knowing a little bit about the rules governing quiver conjunction. Specifically, about addition of two quivers sharing one element (say, A-B and B-C) In which way is determined the dominant element from the quiver, so to speak? Thnaks in advance

0
Q: expected value E(X) of couples that sit together at one table.

Jules20 people, 10 couples, choose between 5 tables, 4 seats each. They choose arbitrarly. What is the E(X) of couples that happen to sit together at one table?

0
Q: Improper Riemann Integral question

Seojun HongSuppose $(a_n)$ is a sequence of real numbers such that $a_n > 0$ for all $n \in \mathbb{N}$ and that there exists $f: [0, \infty) \to \mathbb{R}$ such that $f(n) \leqslant a_n \leqslant f(n-1)$ for all $n \in \mathbb{N}$. Assume that $f$ is a continuous function and decreasing. Also $f > 0$. I...

Words such as question do not add information to titles. Please edit the title so that it better describes the specifics of your question. Do not hesitate to make it longer or include a formula if needed. More tips here. (autocomment)Normal Human 21 secs ago
0
Q: Finding the sum of the infinite series

anonymousHi I am trying to solve this problem: enter image description here I know its a geometric series, but I cannot find the pattern around this.

0
Q: Given the Determinant find the matrix

H. HingI've come across a question that states: Suppose that det(A)=(8+1) Find the R2x2 Matrix(A) How would I find the original matrix?

0
Q: Distribution of Y given X

user3904534Let $X_1$ and $X_2$ be the number on two independent fair-die rolls. Let X be the minimum and Y the maximum of $X_1$ and $X_2$. $(a)$ Find the distribution of $Y|X$ Here is my work: Here is what I know: $P(Y = y| X=x) = \frac{P(Y=y, X=x)}{P(X=x)}$ and $P(X=x)$ can be found by the following e...

0
Q: Matrices inner products

John oliverLet p = −2 + x + 3x^2 and q = 4 − 7x^2 (a) Find ||p||, ||q||, and d(p, q) relative to the standard inner product on P2. (b) Find ||p||, ||q||, and d(p, q) relative to the evaluation inner product on P2 using the sample points x0 = 2, x1 = −1, x2 = 0, and x3 = 1. What is the difference between ...

0
Q: A question on something in Trigonometric Series of Zygmund's.

MathematicalPhysicistOn page 12 it's written that the function ($\delta>0$)$t(x)=1+\cos(x-x_0)-\cos(\delta)$ satisfies the following: $t(x)\ge 1$ in $I$ where $I = (x_0-\delta,x_0+\delta)$ $t(x)>1$ where $I'$ is some interior interval to $I$, and $|t(x)| \le 1$. My question how is it possible that there would be equ...

 
9:02 PM
0
Q: Image of an open set in complex analysis

J.R.Is it true that the image $f(D)$ of any open set $D$ $\subset$ $X$ is open? Here $f$ is a continuous function, and $X$ is a topological space. Can someone explain this to me?

0
Q: Does $Ax = 0$ for nonzero $x$, where $A$ is $n\times n$ over a ring, imply $\det(A) = 0$?

Enjoys MathI know that it's true of $A$ is over a field, but is the same true if just over a ring? How can I prove that it's true?

0
Q: how to find f(x) , given f(x^2)=1/2x

endriI was wondered how to find the function in this equality: f(x^2)=1/2x find f(x) please help me

Short question. Question contains please. how to find f(x) , given f(x^2)=1/2x
 
0
Q: Why was this nice (though closed) question deleted?

amonToday, someone asked Why do we need both Priority and Severity? (10k link) which is clearly on-topic being about software development concepts and all that, but was soon closed as primarily opinion based. That's correct, as it was not an exact duplicate of the much older How do you classify bug s...

 
0
Q: Group theory problem, anout number of subgroups of a group

EklavyaFor a group G,let F(G) denote the collection of all subgroups of G.which of following can occur? A)G is finite,but F(G)is infinite B)G is infinite but F(G)is finite C)G is countable but F(G) is uncountable D)G is uncountable but F(G)is countable option A,B is false,i think option D is also f...

 
9:25 PM
0
Q: Reasons behind lack of feedbacks on a recent question

KolminJust a very small premise: this meta-question is about a specific question, but the take-home lesson can concern many more questions (both from my past and my future behavior as a user). Recently I asked a question concerning a passage from a book, and after some time I put a 150 Bounty (still ...

 
0
Q: Linear Algebra, Linear Transformation problem

Jon RoyMy task is this I am wondering how to go about doing this. Anyone have ant idea? Thanks!

Title contains problem. Short question. Linear Algebra, Linear Transformation problem
0
Q: Equation curves

Kasia PisaniakIntroduce equation curves to the canonical form, finding an appropriate rectangular coordinate system. a) $5x^2+12xy-22x-12y-19=0$ b) $9x^2+24xy+16y^2-230x+110y-475=0$ Could somebody do one task. I don't know how i must do this. Maybe you write in points what I must do? Please help me!

Short title. Question contains please. Equation curves
0
Q: Rolle's theorem problem

DodgeLet f be continuous on an interval [a, b] and differentiable on (a, b) with a derivative that never is zero. Show that f maps [a, b] one-to-one onto some other interval. I can prove that f must map to other intervals, because if for some x1,x2 in [a,b] such that f(x1)=f(x2), then there will be c...

Short title. Title contains problem. Rolle's theorem problem
0
Q: Multivariable calculus questions

user3867875I am new to the site and I have been struggling through two of my homework problems and I have no clue how to do them. The first question is: "Are there any points on surface x^2 -y^2 -z^2 = 1 where the tangent plane is parallel to the plane z= x + y?." I know you have to find the directional ve...

Welcome to Math.SE, user3867875. This site uses MathJax formatting of formulas. More tips here. (from a bot)Normal Human 20 secs ago
0
Q: How to prove the inegality young?

HB khaledGiven $p\in \mathbb{R}$ and $p\geq 2$ . prone that: $$|x+y|^p+|x-y|^p\geq 2(|x|^p+|y|^p)$$

0
Q: The equation $z\sin z=50$

AtaulfoLet $\mathcal S$ be the set of all the solutions of the equation $$z\sin z=50\space\text{with}\space z\in \mathbb C$$ Try to make a description, as complete as possible, of the set $\mathcal S$, in particular prove that $\mathcal S \subset \mathbb R$. What are the elements of $\mathcal S$?

0
Q: Groups and Catalan Numbers

alkabaryI was reading a book about Catalan Numbers (Thomas Koshy Catalan numbers with applications) And I was reading through that example. Find the number of n-element multisets $\{a_1 ,a _2 , . . . , a_n \}$ of elements $a_i \in Z_{n+1}$ such that $a_1 + a_2 + . . . + a_n = 0$, the additive i...

0
Q: wedge of a dual basis

FACEIT I don't understand the final line. why is $\phi_{i_1}(e_{\sigma(i_1)}) = 1$ for example? I thought that $\phi_{i_1}(e_{\sigma(i_1)}) = 1$ if $\sigma(i_1) = i_1$

Short title. Short question. wedge of a dual basis
0
Q: Find the area of the region enclosed by the inner loop of the curve. r = 4 + 8 sin θ

SunnyThe loop is generated for θ in [7π/6, 11π/6]. (this is from setting r = 0). So, A = ∫(7π/6 to 11π/6) (1/2)(4 + 8 sin θ)^2 dθ = ∫(7π/6 to 11π/6) (8 + 32 sin θ + 32 sin^2(θ)) dθ = ∫(7π/6 to 11π/6) (8 + 32 sin θ + 16(1 - cos(2θ))) dθ = ∫(7π/6 to 11π/6) (24 + 32 sin θ - 16 cos(2θ)) dθ = (2...

This site uses MathJax formatting of formulas. More tips here. (autocomment)Normal Human 20 secs ago
0
Q: What is the object of integral problems?

Marygrace We are learning about these in calculus and I can't grasp the concept. Is this similar to the anti-derivative ?

Title contains problem. Short question. What is the object of integral problems?
0
Q: Question on subsequences and convergence

babylonI was asked this question: Prove that $a_n$ converges if and only if: $a_{2n},a_{2n+1},a_{3n}$ all converge I thought this was an easy generic question until I read the hint which said: Note: It is not required that the three sub-sequences have the same limit. This needs to be shown This is w...

Words such as question do not add information to titles. Please edit the title so that it better describes the specifics of your question. Do not hesitate to make it longer or include a formula if needed. More tips here. (autocomment)Normal Human 20 secs ago
0
Q: Taylor expansion of a complex function on a disc

jelenaI need to find the taylor expansion of the complex function z^2/(z-2) on the disc |z|<2 I'm not sure how to start this off, can anyone help me?

0
Q: Moving frame method for ODE

Catch.MeI have read in A Practical Guide to the Invariant Calculus that, but I don't like her style.I need another book on the same topic using moving frame for ODE with the classical notation. Or is there any new technique to look for ODE invariant.

0
Q: properties of a matrix which not negative definite

7779052Is it true that if a matrix is not negative definite, then there exists a row $i$ which the sum of its entries $\sum _j a_{ij}$ is positive?

0
Q: Generalized Birkhoff theorem

zoliLet $A \in \mathbb{R}^{n \times n}$ such that $$ \sum_{k=1}^n |a_{ik}| \leq 1$$ and $$ \sum_{k=1}^n |a_{kj}| \leq 1 $$ for every $1 \leq i, j \leq n.$ As far as I know then there exist scalars $\lambda_i$ such that $\sum_i |\lambda_i| \leq 1 $ such that $$ A = \sum_{i} \lambda_i P_i,$$ whe...

 
10:15 PM
0
Q: Points Awarded to Original Answers in Translated Sites

Edwin DalorzoThis week started the Stackoverflow in Spanish. I went there hoping to start participating and I found a really interesting question. I was about to sit down to start figuring out an answer when I had this feeling that I had already seen this before somewhere else and then I Googled it and found ...

 
0
Q: Prof of Reflexive, symmetric, or transitive relations

ashleybee97Consider the relation R on Z as: ∀m,n ∈Z, mRn ⇔ m − n is odd . Is R reflexive, symmetric, or transitive? What would the proof or counter proof be? Since R is a reflexive since m-n is linear, but I'm not sure how that would work with the proofs.

0
Q: Inner Product - Axioms

matt salem(a) Let u = (u1, u2) and v = (v1, v2). Prove that = 3u1v1 + 5u2v2 defines an inner product on R2 by showing that the inner product axioms hold. (b) What conditions must k1 and k2 satisfy for = k1u1v1 + k2u2v2 to define an inner product on R2? For part a, how do I prove that the inner product ...

0
Q: Summation of differences in exponent

Jakub KawalecCame to this summation during an algorithm analysis problem and any help would be much appreciated:$$\sum_{j=1}^n3^{n-j}$$

 
10:36 PM
0
Q: What is the closed form of \int e^{x^2} \, dx?

user294826How can I do a closed form expansion of \int e^{x^2} \, dx? Please be specific as to which method I must use.

Short question. Question contains please. What is the closed form of \int e^{x^2} \, dx?
0
Q: Automorphism and conjugation

andyLet G be a group and fix h E G show that f:G-G given by f(g)=h^-1gh is an automorphism. Do i need to show that f is bijective first? And then relate it to being an automorphism? A function to be automorphism must be isomorphism=bijective. And automorphism means that G=H

0
Q: Discrete functional notation

kakaMy question has two parts: 1) Is $ H: f \rightarrow \sum\limits_{Y \in \mathbb{R^+}^3}^{} f \cdot \operatorname{Log}_2(f) $ a valid discrete functional ? (H is the entropy defined in here and $f$ is a some discrete function) 2) If yes, then what could be the proper notation of it, if $f$ has...

0
Q: Problem 5 from Section E of Chapter 10 of Pinter's Book of Abstract Algebra

AskerThe problem: Let $a$ and $b$ be commuting elements of a group $G$. Let ord($a$) $=m$ and ord($b$) $= n$; let lcm($m,n$) denote the least common multiple of $m$ and $n$. Prove that there is an element $c$ in $G$ whose order is lcm($m,n$). (HINT: Use the facts that (1) If $m$ and $n$ are relative...

 
0
Q: I found an error in a comment, but have't enough reputation to comment. How can I do to report the error?

Gerardo ZinnoI'm new to stackoverflow. I found an error in an answer here on stackoverflow but I do not have enough reputation to post a comment under it. Is there a way to comment it anyway or do I have to wait and maybe forget about it?

0
Q: Are mass edits removing extensive "thanks" and greeting lines from Q's acceptable for users with 2K Rep and more?

C.O.Salutations, greeting, "thanks", and signatures in questions and answers are discouraged by community consensus. On the help center a text (by Jon Skeet himself) discourages it. We also discourage mass suggested trivial edits to avoid overloading the review queues. The question is - is mass edit...

0
Q: Please vote on the original answer if a dupe- answer only links to the original

Jacob VlijmIt is completely understandable that inexperienced users are not completely aware of how to handle dupes. It is also understandable that a friendly-meaning person posts an answer, (only) linking to another answer, assuming he or she is helpful. Nothing to add, nothing to elaborate, just a link. ...

 
10:59 PM
0
Q: $A$ is compact operator

HysdadLet $A : l_p \to l_p, 1 \le p \le \infty$ be an operator defined by $A\left ( x_n \right ) = \left ( a_n x_n \right )$ where $\left \{ a_n \right \}$ is a bounded real sequence. Prove that $A$ is compact if and only if $\lim_{n \to \infty} a_n =0$. $A$-compact if and only if $A(B)$ is relative...

 
11:16 PM
0
Q: Inverse Laplace Transform of 1/(\sqrt(s+a)+\sqrt(s+b))

Renato Collado TelloI need to calculate the inverse laplace of: $$F(s)=[\frac{1}{\sqrt{s+a}+\sqrt{s+b}}] \qquad \qquad (s>-a\quad ;\quad s>-b;\quad a\neq b) $$

0
Q: Finding equation of line in SVM that separates class

lars Hence, $w1 =w2 =−1/2$ and $b=7/2$ I'm confused by the solution. How do we know that $w_1=w_2$ from the equation $x_1+x_2 = 7$?

 
1
Q: Is [svgz] really worth a dedicated tag?

PeeHaaI was just browsing the newly created tags when I noticed the svgz tag. Which is created and currently being added to "a lot" of questions by a specific user. My spidey sense tells me this is not a very useful tag, because it is simply a gzipped svg file. But I want to double check with people ...

0
Q: How did I know this was an audit by looking at it?

Joshuahttp://stackoverflow.com/review/reopen/10406756 Yeah I passed the audit. It's hard to fail an audit when you know you're looking at it. But I shouldn't know it's audit before I'm done reading the first paragraph of the question. I did. I don't know how I did. I should mark this question CW but ...

 
11:35 PM
0
Q: Invariants of finite groups

AlinaLet $G$ be a finite group acting linearly on $\mathbb{C}^n$ and $\mathbb{C}[X]^G$ be the ring of invariant polynomials. If $G$ is a group generated by reflections, this ring is generated by $n$ algebraically independent polynomials. If $G$ is not a finite reflection group, the ring of invariant p...

Consider adding a tag for a broader subject area to which the question belongs. Some of these tags might fit. (autocomment)Normal Human 21 secs ago
0
Q: Gaussian integers and ideals

Nida). In the ring of Gaussian integers $\Bbb Z[i] = \{a + bi | a, b ∈ Z\}$, describe the ideal $<i>$. b). In the ring of Gaussian integers $\Bbb Z[i]$, show that $2 ∈ J$ where $J$ is the ideal $<i+ 1>$. I'm having trouble doing this one. So far I know that the ideal $<i>=\{(a+bi)i |(a+bi)\in \Bbb...

 
0
Q: How do you search for all the locked questions

PythonMasterA lot of the locked questions in SO that I have seen have been really cool to explore and have a great time laughing and having fun. Apparently, I can't find a way to search for a list of all the locked questions. Is there anyway to search for only locked questions?

 
0
Q: Infinite product equality $\prod_{n=1}^{\infty} \left(1-x^n+x^{2n}\right) = \prod_{n=1}^{\infty} \frac1{1+x^{2n-1}+x^{4n-2}}$

Kay K.Prove the following equation ($|x|<1$) $$\prod_{n=1}^{\infty} \left(1-x^n+x^{2n}\right) = \prod_{n=1}^{\infty} \frac1{1+x^{2n-1}+x^{4n-2}}$$

0
Q: Show that the limit exists

PauloLet f:[0,1]\rightarrow \mathbb{R} (f continuous) show that if x_{m,n}=\frac{m}{n}, with m\in\mathbb{N}, n\in\mathbb{Z}_+, 0\leq m \leq n-1 Then \lim_{n\to\infty} \frac{\sum_{m=0}^{n-1} f(\frac{m}{n})}{n} exists. my teacher showed that the summation is a cauchy sequence. I tried the same but ...

This site uses MathJax formatting of formulas. More tips here. (from a bot)Normal Human 21 secs ago
0
Q: matlab ezplot command

AlanWhen I was trying to use ezplot3('log(y)-y-2*x+2*log(x)') I was supposed something shapes like an ellipse but now got nothing on the screen, wondering what is the reason Cheers

Short title. Short question. Tagged matlab. matlab ezplot command
0
Q: Real Analysis metric space question showing U is contained in M iff the sequence converges

SteveKLet (M, d) be a metric space. Show that U ⊆ M is open if and only if there is no sequence {pn} ∞ 1 in M \ U that converges to some point p ∈ U.

0
Q: Math and science

FarahHeat is a form of energy measured in calories. 1 calorie is the amount of energy required to raise the temperature of 1 gram of water by 1 degree Celsius. How many calories are required to raise the temperature of 78 grams of water by 14 degrees Celsius?

Short title. Math and science
0
Q: prove a1,a2,a3, ...,an are distinct prime numbers where a1=2 and n>1 then a1a2a3...an+1 can be written in the form 4k+3

raolprove a1,a2,a3, ...,an are distinct prime numbers where a1=2 and n>1 then a1a2a3...an+1 can be written in the form 4k+3 I have no ideas how to approach this question. Any hints?

Welcome to Math.SE, raol. Questions tend to get more attention when they have a tag for a broad area of mathematics relevant to the question. Some of these tags might fit. (from a bot)Normal Human 21 secs ago
 
11:57 PM
1
Q: Are answers such as "Clean and Rebuild" NAA?

JALI recently ran into this answer in the First Posts review queue. Please clean your application and build it again. To me, this should be a comment, not an answer, as it is not a concrete answer to the question and more of a suggestion, possibly even asking for clarification from the author....

 
00:00 - 16:0016:00 - 00:00

« first day (29 days earlier)      last day (545 days later) »